Exam Master Heme

Réussis tes devoirs et examens dès maintenant avec Quizwiz!

Defective red cell membrane protein (spectrin) deficiency is seen in which of the following hemolytic anemias? 1 Congenital spherocytosis 2 Thalassemia 3 Sickle cell anemia 4 Paroxysmal nocturnal hemoglobinuria 5 Hemoglobin C disease

1 Congenital spherocytosis

A 20-month-old boy presents for a routine visit. He is eating well and drinking at least a bottle of cow's milk daily for the last several months. His history and physical are normal, except for mild pallor. Urinalysis is normal, and the rest of his laboratory findings are below: Low MCV Elevated RDW What additional finding might you expect in this child? 1 Anisocytosis 2 Drepanocytosis 3 Schistocytosis 4 Elevated bilirubin 5 Increased hemoglobin A2

1 Anisocytosis

A 45-year-old Caucasian man is employed to demolish numerous painted structural steel railroad trestles. The project is scheduled to take 2 years. He is not given protective masks or clothing. Towards the end of the project, he starts to develop abdominal pain, so he sees his family doctor. On questioning during the medical history, he comments that he frequently drops things with his right hand. On physical examination, his lungs are clear to auscultation and no abnormal breath sounds are heard. His cardiac exam is unremarkable. His skin exam is normal other than a pigmentation at the gingivodental margin. His extensor muscles of the wrist and finger are weak. A CBC is done; the results are shown in the chart. RBC 4.5 x 106/µl Mean corpuscular volume 73 cu µ Hematocrit 30 Hemoglobin 10 gm/dL Platelets 250,000/mm3 Comments: Red blood cell stippling is present. Hypochromic red blood cells are present. Where is the toxic element stored? 1 Bones 2 Red blood cells 3 Liver 4 Skin 5 Brain

1 Bones

A 5-year-old African-American girl with sickle cell disease presents due to right leg pain. 2 weeks ago, she began to experience right thigh pain and a slight limp following a playground injury. Upon sustaining the injury, she was taken to a local ER; X-rays were negative for evidence fracture. She was diagnosed with and treated for a leg contusion. Over the past 2 weeks, the pain has become more severe, and she has experienced fever; temperatures have been as high as 102°F, and she has experienced episodic chills. On physical examination, her temperature is 101°F; her right leg is swollen, tender, and erythematous over the anterior aspect of the thigh. The remainder of the PE is unremarkable. CBC with differential reveals a leukocytosis; there is a predominance of neutrophils and a bandemia. What is the most appropriate empiric treatment? 1 Ceftriaxone and Vancomycin 2 Gentamicin and Vancomycin 3 Nafcillin and a Fluoroquinolone 4 Nafcillin and Vancomycin 5 Vancomycin and a Fluoroquinolone

1 Ceftriaxone and Vancomycin

A 19-year-old woman presents with weakness and increased fatigue over the past several months. Laboratory studies reveal haemoglobin of 9.7g/dL with a haematocrit of 32%. Which of the following results would indicate an iron deficiency anemia? 1 Increased TIBC, decreased serum ferritin, decreased serum iron 2 Decreased TIBC, decreased serum ferritin, decreased serum iron 3 Increased TIBC, increased serum ferritin, increased serum iron 4 Increased TIBC, increased serum ferritin, decreased serum iron 5 Decreased TIBC, increased serum ferritin, decreased serum iron

1 Increased TIBC, decreased serum ferritin, decreased serum iron

A 2-year-old boy presents for routine checkup. He is a known case of sickle cell disease and is on penicillin prophylaxis. The child is asymptomatic and appears healthy. He has received a regular schedule of immunizations to date. What additional vaccination needs to be administered in his case? 1 Influenza, pneumococcal, and meningococcal vaccine 2 DTaP and Hepatitis B vaccine 3 IPV and varicella vaccine 4 Tdap, MMR, and Hepatitis B vaccine 5 Only influenza vaccine

1 Influenza, pneumococcal, and meningococcal vaccine

A 34-year-old woman presents with fatigue, headache, shortness of breath, cravings, heartburn, and dizziness upon rising from a chair. The patient denies pregnancy and recent illness; she is taking no medications. Upon physical exam, her skin is pale; her nails are brittle, and she has a blood pressure of 108/76 mm Hg. You order a blood test. What is the most likely diagnosis? 1 Iron deficiency anemia 2 Folic acid deficiency 3 Pernicious anemia 4 Hemolytic anemia 5 Sickle cell anemia

1 Iron deficiency anemia

A 45-year-old Caucasian man is employed to demolish numerous painted structural steel railroad trestles. The project is scheduled to take 2 years. He is not given protective masks or clothing. Towards the end of the project, he starts to develop abdominal pain, so he sees his family doctor. On questioning during the medical history, he comments that he frequently drops things with his right hand. On physical examination, his lungs are clear to auscultation and no abnormal breath sounds are heard. His cardiac exam is unremarkable. His skin exam is normal other than a pigmentation at the gingivodental margin. His extensor muscles of the wrist and finger are weak. A CBC is done; the results are shown in the chart. TEST RESULTS REFERENCE RANGE RBC 4.5 x 106/µl 4.5 - 5.7 x 106/µl (male) 3.9 - 5.0 x 106/µl (female) Mean corpuscular volume 73 cu µ 80 - 100 cu µ Hematocrit 30 40 - 50 (male) 36 - 44 (female) Hemoglobin 10 gm/dL 13.8 - 17.2 gm/dL (male) 12.1 - 15.1 gm/dL (female) Platelets 250,000/mm3 130,000 - 400,000 mm3 Comments: Red blood cell stippling is present. Hypochromic red blood cells are present. What is the most likely diagnosis? 1 Lead poisoning 2 Chronic mercury poisoning 3 Chronic carbon monoxide poisoning 4 Silicosis 5 Berylliosis

1 Lead poisoning

A 57-year-old woman presents with pain and swelling in her left leg. Her chart shows a history of osteoarthritis of the knees, mild hypertension, and type II diabetes mellitus, controlled on medication. She was well until one week ago, when she noted a bulge behind her left knee. Yesterday she woke up with pain, redness and swelling in her calf, and stayed in bed most of the day, with her leg propped up on a pillow. She has had no fever or chills, no new pain or swelling in her right leg, no shortness of breath, and does not otherwise feel ill. She denies recent travel or immobility. Vital signs are within normal limits. Significant on her physical exam is a red, warm and swollen left calf with trace pitting. There is no swelling in the thigh or toes, feet are warm and pedal pulses are intact. The left calf is tender to palpation posteriorly, and is one centimeter larger than the right calf on measurement. The rest of her exam is unremarkable. A sensitive assay D-dimer blood test is negative. Based on this information, the likelihood of deep vein thrombosis is 1 Low, because the patient has a low risk for DVT given the negative sensitive D dimer test. Ultrasound can be considered to further evaluate the knee 2 Low, but the patient is at high risk for DVT. Order doppler ultrasound studies and if negative, repeat in one week 3 High, as indicated by swelling and tenderness on exam, and immediate anticoagulation should be ordered 4 High, because D-dimer tests have a poor negative predictive value. Order compression ultrasound 5 Low , because symptoms of heat, swelling and redness are more consistent with erysipelas. Order dicloxacillin 500 mg four times a day

1 Low, because the patient has a low risk for DVT given the negative sensitive D dimer test. Ultrasound can be considered to further evaluate the knee

A 60-year-old man presents to the office for routine follow-up of hypertension. A CBC reveals: WBC 12.0 x 103/mm3 4.5 - 11 RBC 7.0 x 106/μl 4.6 - 6.2 (male) Hematocrit 60 % 40 - 54 (male) MCV 68 μ3 80 - 100 RDW 14.9 % 11.6 - 14.6 Platelets 550,000 mm3 150,000 - 450,000 On examination, blood pressure is 160/88 mm Hg, pulse is 86/min, temperature 97.6°F, and respirations 18/minute. His skin appears ruddy and the tip of the spleen is palpable. Pulse oximetry reveals a saturation of 99%. What distinguishing finding would you expect on history or physical exam? 1 Pruritis after a hot shower 2 Clubbing of the nails 3 Visual disturbances 4 Easy bruising 5 Palpable renal mass

1 Pruritis after a hot shower

A 70-year-old man presents with weakness in his arms and legs as well as disorientation. You suspect pernicious anemia, but he tells you that he has been taking vitamin B12 capsules. You decide to give him injections of vitamin B12 to see if this alleviates the symptoms. He returns to your office a few weeks later and says that he no longer has the weakness or disorientation. This confirms your suspected diagnosis. In this case, what caused the man's condition? 1 Reduced synthesis of intrinsic factor 2 Reduced capacity to synthesize vitamin B12 3 Inability of vitamin B12 to bind folic acid 4 Absence of cobalt from the diet 5 Lack of vitamin B12 in the diet

1 Reduced synthesis of intrinsic factor

A 28-year-old woman presents with heavy menstrual bleeding; she soaks through a pad every hour or so. She also has a 2-week history of fatigue and bleeding from her gums. Physical examination reveals a pale woman with an enlarged spleen and diffuse petechiae. A complete blood cell count shows leukocytosis (WBC 10,000), with 29% blasts as well as the presence of Auer rods, anemia (hemoglobin 7.9 g/dL), and thrombocytopenia (platelet count 25,600). A bone marrow biopsy is positive for AML. What will be your next step in the management of the disease? 1 Remission induction 2 Prophylactic antibiotics 3 Consolidation therapy 4 Red cells transfusion 5 Gonadotropin-releasing hormone agonists

1 Remission induction

A 64-year-old man presents to his PCP for the first time in 4 years with a recent history of fatigue with pallor and worsening back pain in the lumbar region for the past 3-4 months. He denies any injury to his back or similar pain in the past. He had a colonoscopy 3 months prior, and 2 benign polyps were removed without other abnormalities. At that time, he also had a PSA level of 3.1 ng/ml. On examination, BP: 143/92 mmHg, P: 75 bpm and regular, Resp: 16/minute, Temp: 97.6 degrees F. Skin exam reveals marked pallor without lesions or bruising. Musculoskeletal exam reveals bony tenderness along lumbar vertebrae without paraspinal muscle tenderness to palpation. Decreased range of motion of spine noted on examination due to pain elicited during examination. The remainder of the examination including digital rectal exam, focal neurological, pulmonary, and cardiac exams are within normal limits. Which of the following tests will be most helpful in making your diagnosis? 1 Serum protein electrophoresis 2 Complete blood count with differential and platelets 3 Serum calcium level adjusted for serum protein 4 Total serum protein 5 Urine albumin

1 Serum protein electrophoresis

A 26-year-old woman presents with fatigue, malaise, headaches, blurred vision, and tinnitus. She denies any chest pain, abdominal pain, nausea, vomiting or diarrhea. She has not noticed any dark stools or abnormal urine. On exam she has a temperature of 100.3 F, pulse rate 102/min, BP 130/84, and SPO2 is 99%. Chest and abdominal exam are unremarkable, but the skin on her legs show ecchymoses. Labs show Hb 8.8g/dl, WBC 11,000, and platelets 65,000. PT and PTT are normal. BUN is 16, creatinine is 1.1, and AST/AST are normal. UA shows 12 RBC with 5 WBC. Peripheral smear reveals fragmented red cells and LDH is 620 U/L. What is the most likely diagnosis? 1 Thrombotic thrombocytopenic purpura (TTP) 2 Hemolytic-uremic syndrome (HUS) 3 Idiopathic thrombocytopenic purpura (ITP) 4 Antiphospholipid antibody syndrome (APS) 5 Disseminated intravascular coagulation (DIC)

1 Thrombotic thrombocytopenic purpura (TTP)

A 63-year-old man presents with bleeding gums and a general feeling of malaise. He states that he has been running a low-grade fever for the past few days, feels fatigued, and has bone and joint pain. He thought that he had the flu, but became concerned when his gums began to bleed. Physical examination reveals a pale individual, mild petechiae on the extremities, gingival hyperplasia, and a palpable spleen. A CBC reveals pancytopenia and blasts with Auer rods on the differential. What is the most likely diagnosis? 1 Chronic lymphocytic leukemia (CLL) 2 Acute myelogenous leukemia (AML) 3 Multiple myeloma 4 Chronic myelogenous leukemia (CML) 5 Polycythemia vera

2 Acute myelogenous leukemia (AML)

A 44-year-old woman presents for her annual exam. Findings at the physical exam are BP-120/80 mmHg, HR - 70 beats/min, pale mucous membranes, and increased uterine size. The patient also presents with menorrhagia of 2 years duration. Her blood test shows microcytic hypochromic red blood cells (RBCs). Further exams show low serum iron, increased total iron-binding capacity (TIBC), and low ferritin. What is the most appropriate diagnosis for this patient? 1 Anemia of chronic disease 2 Anemia secondary to iron deficiency 3 Beta thalassemia 4 Megaloblastic anemia 5 Sideroblastic anemia

2 Anemia secondary to iron deficiency

A patient in the hospital for a wound infection is referred to you for consult. After 3 days of antibiotic therapy, the patient has gone from normal RBC results to a normocytic anemia with a slight macrocytosis (see image). The reticulocyte count is decreased. More significant is the development of a pronounced pancytopenia. A review of the charts reveals that this pancytopenia was not present upon admission. The patient has been on chloramphenicol therapy. What is the most likely diagnosis? 1 Thalassemia 2 Aplastic anemia 3 Megaloblastic anemia 4 Fragmentation syndrome 5 Sickle cell anemia 6 Paroxysmal nocturnal hemoglobinuria (PNH)

2 Aplastic anemia

A 48-year-old previously healthy African American woman was involved in a severe motor vehicle accident, sustaining multiple injuries. She was stabilized in the Emergency Department, but is now bleeding extensively from her laceration sites, her IV catheter site, and from mucous membranes. Laboratory results show thrombocytopenia, fragmented red blood cells, and low fibrinogen levels. What is this patient's most likely diagnosis? 1 Factor V deficiency 2 Disseminated intravascular coagulation 3 Factor XI deficiency 4 Protein C and S deficiency 5 Acute idiopathic thrombocytopenia purpura

2 Disseminated intravascular coagulation

A 22-year-old man presents to his physician with swollen lymph nodes in the right axilla. He notes that he develops pain in the area after drinking alcohol. He has been feeling fatigued for the last few weeks and has lost weight without trying. Examination confirms lymphadenopathy in the right axilla. Biopsy of the region shows the presence of Reed Sternberg cells. What is the treatment of choice at this time? 1 Lenalidomide and prednisone 2 Doxorubicin, bleomycin, vinblastine, and dacarbazine 3 Radiation therapy 4 Stem cell transplant 5 Observation

2 Doxorubicin, bleomycin, vinblastine, and dacarbazine

A 63-year-old Hispanic-American man presents with a 3-month history of chronic intermittent diarrhea with 5 - 6 watery stools per day; some are tinged with blood. During this period, he has had progressive fatigue and intolerance to exercise; he has lost 15 pounds (6% of body weight). Vital signs are within normal ranges. On physical examination, there is noticeable pallor. Heart sounds are regular and of normal frequency, both lung fields are clear to auscultation, the abdomen is soft and diffusely tender to palpation, particularly in the right quadrants. Rectal examination is painless, and an enlarged prostate is palpated, but there are no masses or blood. Anoscopy shows grade I hemorrhoids. There is no peripheral edema. A complete blood count shows the following: Ht 30% Hb 9.4 g/dL MCV 77 fl MCHC: 25 g/dL WBC 8.600 /mm3. w/ normal differential, platelets 460.000 /mm3; iron 56 µg/dL ferritin 5.5 µg/dL What is the most appropriate next step in management? 1 Fecal occult blood test 2 Full colonoscopy 3 Barium enema 4 Sigmoidoscopy 5 Computed tomography of the abdomen

2 Full colonoscopy

A 22-year-old African-American man presents with excruciating pain in his back for the past 2 days. He denies any history of trauma. He gives a history of similar pains in the past; however, he is not on any medications. His older sister has a similar illness. On examination, his HR is 110 beats/minute, and he has a tinge of jaundice. What laboratory result would you expect after analyzing his blood? 1 Hemoglobin of 16 g/dl 2 Hematocrit of 21% 3 Microcytic red cells 4 Reticulocytosis of 2% 5 Bilirubin of 1 mg/dl

2 Hematocrit of 21%

A 29-year-old African-American woman of Central African descent, who recently immigrated to the United States, presents for her 1st prenatal visit. Her CBC and red cell indices are as follows: RBCs 4.9 x 106/mL Hemoglobin 12.9 g/dL HCT 42% MCV 85 fL MCH 26 pg MCHC 31 g/dL WBCs 9.0 x 103/mL Platelets 200 x 103/mL Morphologic review of the peripheral blood smear shows normal erythroid, platelet, and white blood cell morphology. What test should next be ordered? 1 Free erythrocyte protoporphyrin 2 Hemoglobin electrophoresis 3 Sickle cell test 4 Solubility test for hemoglobin S 5 Stainable bone marrow iron

2 Hemoglobin electrophoresis

A 76-year-old male was diagnosed with early bladder carcinoma and received intravesicular Mitomycin C as treatment. He received 3 doses a week and was in the 15th dose when he started feeling unwell with increasing malaise, nausea, vomiting, swelling of face, and pedal edema. He was admitted to the hospital and found to have a BP of 176/102, PR 80/min, SPO2 92%, and temperature of 99 F. Labs showed a Hb of 7.7g/dl, WBC 12,000, and platelets 20,000. Peripheral smear revealed red cell fragmentation with schistocytes and low platelets. Serum creatinine was 4.5 and BUN 56. Indirect bilirubin was 4.5, LDH was 500, and AST and ALT were normal. Prothrombin time was normal, with an INR of 0.9, whereas PTT was 29. UA showed 20 RBCs, 35 WBCs, and granular casts. He remained alert and oriented, without any neurological deficits. Based on the above what is the most likely diagnosis? 1 Thrombotic thrombocytopenic purpura (TTP) 2 Hemolytic-uremic syndrome (HUS) 3 Idiopathic thrombocytopenic purpura (ITP) 4 Antiphospholipid antibody syndrome (APS) 5 Disseminated intravascular coagulation (DIC)

2 Hemolytic-uremic syndrome (HUS)

A 44-year-old Caucasian woman is admitted to the hospital with a severe nosebleed. The patient states that there was no history of trauma and she never has had nosebleeds before. She also reports a history of upper respiratory infection (URI) symptoms 1 week ago. The patient's blood work is notable for a platelet count of 10,000/cmm. A presumptive diagnosis of idiopathic thrombocytopenic purpura (ITP) is made, and the patient is treated with steroids. The patient's platelet count is refractory and a splenectomy is performed. After a few months, what would you expect to see in this patient's peripheral blood smears? 1 Burr cells 2 Howell-Jolly bodies 3 Hypersegmented neutrophils 4 Schistocytes 5 Spherocytes

2 Howell-Jolly bodies

A 25-year-old Scandnavian male presents with terrible discomfort after weeks of working in a lead-zinc mine. The symptoms include indigestion, belching, and diarrhea. His tongue is red and sore. He reports easy fatigability with minimal exertion, accompanied with difficulty breathing. Physical examination reveals high blood pressure and tachycardia. In addition, you observe splenomegaly, and pallor of the skin and mucus membrane. Peripheral blood smear examination indicates large oval RBCs, fragmented cells, hypersegmented neutrophils, and poikilocytosis. Lab values reveal: Hb, 9 g%; RBC count, 3.5 millions/cu mm of blood; PCV, 39%; ESR,15 mm at the end of first hour ; MCV, 110u3; Serum, Vit B12, 160 p mol/l, and non responsiveness of Histalog test. What is the most likely diagnosis? 1 Dietary deficiency of Vit B12 2 Intrinsic factor deficiency 3 Defect in the site of absorption of vitamin B12 4 Folic acid deficiency 5 Iron deficiency

2 Intrinsic factor deficiency

A 66-year-old woman presents with a several-month history of tingling and numbness in her legs. She lives with her daughter, who is also present in the room. The daughter tactfully tells the doctor that her mother has been increasingly irritable of late; she has also become forgetful. She seems fatigued and generally dull. Past medical history includes hypertension, controlled by amlodipine 10 mg daily, and osteoarthritis of her knees and hips. She also has a history of hypothyroidism, for which she is on thyroxine 75 micrograms daily. She does not smoke or drink excessively. Physical exam reveals BP of 130/82 mm of Hg, pulse 76/min, SPO2 93%, and temperature of 98.2 F. Her skin has a sallow appearance, arthritic changes are apparent in her knees bilaterally, and her tongue is shiny; her lungs are clear, and her abdomen is benign. Neurological examination shows loss of vibration and position sense, as well as mild ataxia. She is alert and oriented to time, place, and person. Labs are ordered, which reveal Hb of 9g/dl, MCV 100, WBC 5600, platelets 310,000, and peripheral smear with hypersegmented polymorphonuclear leukocytes. Serum iron studies are normal. Thyroid function studies are normal. Based on the above clinical picture, what is your provisional diagnosis? 1 Anemia of chronic disease 2 Pernicious anemia 3 Thalassemia 4 Iron deficiency anemia 5 Aplastic anemia

2 Pernicious anemia

A 77-year-old man presents with left-sided chest pain, headaches, night sweats, a burning sensation in his hands and feet, and shortness of breath. 2 days ago, he began having headaches; his symptoms have progressively worsened. On exam, the patient is a thin, ill-appearing man. His spleen is palpated 8 cm below the costal margin, and the liver 5 cm below the costal margin. Laboratory analysis shows that his red blood cell mass is elevated, and he has a decrease in plasma volume. White blood cells and platelets are within normal limits. Appearance of the red blood cells is normal. Arterial blood gas analysis shows an O2 saturation of 94% on room air. What is the most likely cause of this patient's symptoms? 1 Chronic myelogenous leukemia 2 Polycythemia vera 3 Thrombocytopenia 4 Iron-deficiency anemia 5 Aplastic anemia

2 Polycythemia vera

A 23-year-old woman presents with fatigue and the recent onset of a yellowing of her skin. Her physical examination is remarkable for the presence of splenomegaly. Laboratory results are as follows: Laboratory results are as follows: TEST RESULTS hematocrit 38 hemoglobin 12.6 gm/dL RBC 3.3 x 106/µl MCHC 37 platelets 258,000/mm3 WBC 5.4 x 103/mm3 neutrophils 57 lymphocytes 37 monocytes 4 eosinophils 2 basophils 0 Comments: Red blood cells are spherocytes. Red blood cells demonstrate increased osmotic fragility. Where is the primary pathology? 1 Heme 2 RBC membrane 3 α globin chain 4 β globin chain 5 Spleen

2 RBC membrane

A 22-year-old Asian man is presenting to a hematology clinic for the first time to seek management for his alpha thalassemia (hemoglobin H disease type). He has a strong family history of the disorder and was previously being managed by a different hematologist. Today, he receives a thorough history and physical examination, as well as various blood tests. What physical examination finding is most likely to be present? 1 Hepatomegaly 2 Splenomegaly 3 Decreased vibratory sensation 4 Positive Murphy's Sign 5 Retinopathy

2 Splenomegaly

A 35-year-old African-American woman on dialysis has had a 4-month history of hemoglobin of 9 g/dL, despite erythropoietin and intravenous iron therapies. Diagnostic studies have shown the following: Transferrin saturation 30% Ferritin 400 ng/mL Parathyroid hormone level 700 pg/mL (normal for stage 5 kidney disease 150-300 pg/mL) Serum aluminum Normal Total white blood cell count 5x10 3/mm3 Platelet count 200x10 3/mL What therapy should be used in addition to erythropoietin and iron to treat anemia in this patient? 1 Thyroid replacement hormone 2 Vitamin D therapy 3 Oral iron replacement 4 Intravenous iron therapy 5 Vitamin C

2 Vitamin D therapy

A 9-month-old female from Cyprus presents to her pediatrician. She is suffering from failure to thrive, diarrhea, and recurrent fever. Upon examination, the physician notes that the infant is pale and has an enlarged abdomen due to hepatosplenomegaly. Blood analysis reveals a decreased mean corpuscular volume. Mean corpuscular hemoglobin and hemoglobin are also decreased at 15 pg and 6 g/dl, respectively. A peripheral smear shows microcytic, nucleated red blood cells; otherwise, the smear shows normal morphology and a few target cells. What is the most likely diagnosis? 1 α-Thalassemia 2 β-Thalassemia 3 Hemophilia A 4 Sickle cell anemia 5 Von Willebrand disease

2 β-Thalassemia

Patients with ITP (immune thrombocytopenic purpura) are at risk of spontaneous internal bleeding when the platelet count goes below what level? 1 50,000 2 30,000 3 10,000 4 2,000

3 10,000

A 38-year-old woman of Northern European origin is brought to the emergency room by her relatives, who report abnormal behavior. The patient denies the accusation and complains of numbness and a tingling sensation in both her hands and feet (gloves and stockings) and recurrent diarrhea. Physical examination showed an atrophic tongue (glossitis), and a neurologic examination revealed a loss of her sense of vibration and fine touch. Endoscopic examination showed atrophic gastric mucosa and fasting achlorhydria. What finding aligns in favor of a diagnosis of pernicious anemia in this patient? 1 Gastric parietal cell antibody assay is negative 2 Peripheral blood film shows characteristic microcytic hypochromic red blood cells 3 Bone marrow shows megaloblastic erythropoiesis 4 Negative Schilling test 5 Hyperchlorhydria

3 Bone marrow shows megaloblastic erythropoiesis

A 55-year-old man presents with a 1-week history of fatigue, night sweats, and low-grade fever. On physical examination, you note a palpable spleen. You order a CBC, and the results indicate a white blood count of 105,000 cells/mcL with a left shift of the myeloid series. The red blood cell count and morphology are unremarkable, and he has a slightly elevated platelet count. To help confirm your suspicions, you order genetic studies, and the results come back with the BCR/ABL gene detected. What is the most likely diagnosis? 1 Chronic lymphocytic leukemia 2 Acute lymphocytic leukemia 3 Chronic myelogenous leukemia 4 Acute myelogenous leukemia 5 Polycythemia vera

3 Chronic myelogenous leukemia

A 65-year-old woman presents with pain in the back, chest, and at the right seventh rib. She appears to be quite pale, and she admits to fatigue. These symptoms have come on gradually over a period of several weeks, with the back pain becoming the reason for consulting the physician. Physical examination reveals localized tenderness at the spine of T8 and ribcage with tenderness at the right seventh rib. CBC shows a normochromic, normocytic anemia with hemoglobin of 8 g/dL. Peripheral smear shows marked rouleaux formation with normal platelet and white cell counts and morphology. Serum chemistry results include calcium elevation to 12.2 mg/dL with normal alkaline phosphatase. A dipstick urinalysis shows proteinuria. A bone marrow aspirate was dry. The bone marrow biopsy is pending. Plain film X-rays of the chest show a fracture of the right seventh rib and compression of the eighth vertebra? What is the most likely diagnosis? 1 Chronic lymphocytic leukemia (CLL) 2 Chronic Myelogenous Leukemia (CML) 3 Hairy-cell leukemia (HCL) 4 Multiple myeloma 5 Myelofibrosis

3 Hairy-cell leukemia (HCL)

A 28-year-old female, with no particular past medical history, notices a rash on both her thighs as she is changing clothes one morning. She delivered a baby by C-section approximately 3 months ago. Since then she has had a fairly unremarkable post partum course, except for a respiratory infection 3-4 weeks ago, for which she received sulfamethoxazole/trimethoprim (Bactrim DS). She is no longer nursing. On exam her temperature in 98.4, pulse rate 80/min, BP 110/70, and SPO2 92%. Oral exam reveals two small hematomas on her tongue and one on the inside of her lower lip. Her thighs show diffuse petechiae bilaterally. Chest and abdominal, as well as neurological, exam is normal. Labs show Hb 12g/dl, WBC 7400, and platelets 6000. MCV is 92, PT/PTT are normal, BUN 14, creatinine 0.9, AST/ALT normal, and UA shows 2 WBC and 0 RBC. Peripheral smear was non contributory. Patient is empirically started on steroids at 80mg daily, and platelets are checked every third day. The number goes up to 96,000 over the next few weeks. In all probability, the patient has: 1 Thrombotic thrombocytopenic purpura 2 Hemolytic-uremic syndrome 3 Idiopathic thrombocytopenic purpura 4 Antiphospholipid antibody syndrome 5 Disseminated intravascular coagulation (DIC)

3 Idiopathic thrombocytopenic purpura

A 3-year-old boy in your practice has sickle cell anemia and is also followed by a pediatric hematologist. He has been admitted to the hospital with a history of three days of fever ranging from 38 to 38.5 degrees Centigrade, a cough, and increased work of breathing. Despite aggressive therapy, the patient continues to have a decline in oxygen saturation level. A decision is made to administer hydroxyurea and to perform an exchange transfusion. What effect would hydroxyurea be expected to have? 1 Decrease cell adherence 2 Improve bacterial phagocytosis 3 Increase fetal hemoglobin 4 Promote vasodilation 5 Reverse dehydration of sickled red blood cells

3 Increase fetal hemoglobin

A 60-year-old male presents to the office for routine follow-up of hypertension. A CBC on the front of the chart reveals: WBC 12.0 x 103/mm3 4.5 - 11 RBC 7.0 x 106/μl 4.6 - 6.2 (male) Hematocrit 60 % 40 - 54 (male) MCV 68 μ3 80 - 100 RDW 14.9 % 11.6 - 14.6 Platelets 550,000 mm3 150,000 - 450,000 As you begin your workup of erythrocytosis, what are the 3 major criteria required to make the clinical diagnosis of Polycythemia Vera? 1 Increased RBC mass, increased WBCs, increased Platelets 2 Increased RBC mass, normal O2 saturation, low Erythropoietin level 3 Increased RBC mass, normal O2 saturation, splenomegaly 4 Increased RBC mass, splenomegaly, normal Erythropoietin level 5 Increased RBC mass, increased Hgb, increased Hct

3 Increased RBC mass, normal O2 saturation, splenomegaly

A 32-year-old woman starts to notice that she is constantly tired and run down. She also has started to notice that she is having difficulty swallowing. She sees you in your office with her complaints. On physical examination, you immediately notice that she is pale. You also notice that she has angular stomatitis and atrophic glossitis. Laboratory results are as follows: TEST RESULTS REFERENCE RANGE Hematocrit 30 40-50 (male) 36-44 (female) Hemoglobin 10.1 gm/dL 13.8-17.2 gm/dL (male) 12.1-15.1 gm/dL (female) RBC 3.8 x 106/μl 4.5-5.7 x 106/μl (male) 3.9-5.0 x 106/μl (female) Platelets 225,000/mm3 130,000 - 400,000/mm3 Mean corpuscular volume 75 cu μ 80-100 cu μ WBC 4.8 x 103/mm3 4.0-11 x 103/mm3 Neutrophils 67 40-75 % Lymphocytes 25 15-45 % Monocytes 4 1-10 % Eosinophils 3 1-6 % Basophils 1 0-2 % Comments: hypochromia is present Of the following, which is the most likely cause of her anemia? 1 Vitamin B12 deficiency 2 Folic acid deficiency 3 Iron deficiency 4 Reticulocytosis 5 Bone marrow failure

3 Iron deficiency

An 18-year-old woman needs to be scheduled for an elective cholecystectomy and was assessed to have a possible bleeding disorder. She reports frequent bruising, episodic epistaxis, bleeding gums, and heavy menstrual cycles for a few years. She takes no medications and has no medical illnesses. What lab results will support a diagnosis of von Willebrand's disease in this patient? 1 Low hemoglobin and hematocrit levels 2 A prolonged prothrombin time 3 Prolonged partial thromboplastin and bleeding times 4 Thrombocytopenia 5 Reduced factor IX levels

3 Prolonged partial thromboplastin and bleeding times

Deficiency of what factor may predispose a person to recurrent thrombosis? 1 Platelet deficiency 2 Factor VIIIC deficiency 3 Protein C deficiency 4 von Willebrand factor deficiency 5 Factor VII deficiency

3 Protein C deficiency

A patient with nephrotic syndrome and normal creatinine clearance, presents with sudden onset of left flank pain, and hematuria. Laboratory tests show a rapid deterioration of renal function. Ultrasound demonstrates the left kidney to be larger than the right. Which of the following is the most likely diagnosis? 1 Acute pyelonephritis 2 Renal artery occlusion 3 Renal vein thrombosis 4 Nephrolithiasis 5 Ruptured renal cyst

3 Renal vein thrombosis

An 11-month-old African-American boy presents with an acute onset of anorexia, irritability, unexplained bruising, and jaundice. On examination, you note pale conjunctivae, icteric sclerae, and splenomegaly. Laboratory studies reveal decreased hemoglobin and hematocrit and a significantly elevated reticulocyte count. Hemoglobin electrophoresis reveals the presence of hemoglobin S. What is the most likely diagnosis in this patient? 1 Alpha-thalassemia 2 Beta-thalassemia 3 Sickle cell disease 4 Glucose-6-phostphate dehydrogenase (G6PD) deficiency 5 Hepatitis B

3 Sickle cell disease

A 23-year-old woman presents with fatigue and the recent onset of a yellowing of her skin. Her physical examination is remarkable for the presence of splenomegaly. Laboratory results are as follows: Comments: Red blood cells are spherocytes. Red blood cells demonstrate increased osmotic fragility. What would be an appropriate treatment for this patient? 1 Hemodialysis 2 Phlebotomy 3 Splenectomy 4 B12 injection 5 Iron supplements

3 Splenectomy

A 6-year-old girl is seen by her pediatrician because she has had frequent and prolonged bilateral nosebleeds. Her parents are concerned because there is a family history of a bleeding disorder. Her father, paternal grandmother, and a paternal aunt are all affected. Bleeding time is increased but platelet count and clotting time are normal. Ristocetin cofactor activity is found to be increased. Coagulation factor assays show slight decrease in factor VIII but normal factor IX levels. What is the most probable diagnosis for this family? 1 Hemophilia A 2 Hemophilia B 3 Von Willebrand disease 4 Factor XIII deficiency 5 Primary thrombocythemia

3 Von Willebrand disease

An 18-year-old woman presents with chronic fatigue and menorrhagia. Menorrhagia has been present since her first menstrual cycle, but it has recently become worse; she sometimes uses 20 tampons per day. About a year ago, she started using contraceptive pills but is now considering stopping using them because of migraine-like headaches. Her headaches are sometimes so severe that she has to take aspirin or other pain killers several times a day. The rest of her past medical history is unremarkable. Physical examination reveals pale skin and mucosa, pulse rate of 100 beats per minute, and a systolic ejection murmur 1/3 intensity over the precordium. Laboratory findings are: white blood count 9 K, hemoglobin 10, platelet count 250 K, normal prothrombin time, slightly prolonged partial thromboplastin time, and normal fibrinogen. Her bleeding time is prolonged. Blood smear shows microcytic, hypochromic anemia. What is the most likely diagnosis? 1 Endometriosis 2 Ventricular septal defect 3 Von Willebrand disease 4 Hemolytic uremic syndrome 5 Hemophilia A

3 Von Willebrand disease

A 5-year-old child developed fever, hemorrhages, and repeated bacterial infections. Her white blood cell count was 50,000/µL. A smear of the patient's peripheral blood is shown in the image. What is the most likely diagnosis? 1 acute myelogenous leukemia 2 chronic myelogenous leukemia 3 acute lymphocytic leukemia 4 chronic lymphocytic leukemia 5 hairy cell leukemia

3 acute lymphocytic leukemia

A 5-year-old Hispanic boy with acute lymphoblastic leukemia presents with fever and rash. He is currently on induction chemotherapy that includes dexamethasone. He was exposed to a friend with varicella 2 weeks ago; he has never had the varicella vaccination and his mother cannot recall that he ever had a varicella infection. Blood tests prior to initiation of chemotherapy showed the absence of antibody to the varicella virus. He now has 10 small vesicles and several red macules on his face and chest. Temperature is 38.5° C. Exam is otherwise normal. What is the most appropriate immediate treatment for this patient? 1 Observation 2 Varicella-zoster immune globulin 125 units/10 kg IM 3 Varicella vaccine 0.5 mL IM 4 Acyclovir 500 mg/m2 IV every 8 hours 5 Acyclovir 200 mg/m2 PO 5 times a day

4 Acyclovir 500 mg/m2 IV every 8 hours

A 40-year-old Caucasian man presents with rapidly enlarging skin lesion on his right shoulder. Upon further questioning, he admits to long-term outdoor exposure as required by his occupation as a construction worker. He further notes that the lesion has been changing colors over the last 2 months, first beginning as solid black then evolving to light brown and red, but he denies any bleeding or pain to the area. Physical exam confirms a 7 mm macular lesion with irregular borders and a mottled color pattern on the right shoulder. What risk factor serves as the best predictor of metastatic potential of this lesion? 1 Lesion location 2 Presence of ulceration 3 Color variegation 4 Breslow thickness 5 Serum LDH levels

4 Breslow thickness

A 29-year-old African-American woman and a 31-year-old African-American man present to your office for premarital screening for sickle cell disease. The woman's complete blood count (CBC) and red cell indices are as follows: RBC 4.9 X 106/μL Hemoglobin 12.9 gm/dL HCT 43% MCV 87 fL MCH 26 pg MCHC 30 gm/dL WBC 9.0 x 103/μL Platelets 250 x 103 The man's CBC and red cell indices are as follows: RBC 5.0 X 106/μL Hemoglobin 13.0 gm/dL HCT 43% MCV 85 fL MCH 26 pg MCHC 31 gm/dL WBC 8.0 x 103/μL Platelets 220 x 103/μL Morphologic review of the peripheral blood smear from both candidates shows normal red blood cell, platelets, and white blood cell morphology. What is the next step in their management? 1 Reassurance as the woman's RBC count and indices are normal 2 Both candidates require treatment for sickle cell anemia with hydroxyurea 3 Candidates need to make certain life adjustments, but no therapy is necessary 4 Candidates need to be evaluated by hemoglobin electrophoresis 5 Repetition of CBC and the red cell indices in the woman

4 Candidates need to be evaluated by hemoglobin electrophoresis

A 58-year-old man presents with a 4-month history of worsening fatigue. The patient has noted "enlarged nodes" along the right side of his neck; he has never noticed these nodes before and states they are bothersome due to having so many; they also seem to be getting bigger. Physical examination reveals multiple cervical lymphadenopathies involving both the posterior cervical and the superficial cervical lymph nodes on the right side. Labs obtained the day of presentation include abnormalities, including a WBC count of 250,000/mm3 (normal: 5,000 - 10,000 mm3), lymphocytes being 77% of circulating WBCs (normal: 20 - 40%), and a smear revealing small but mature-appearing lymphocytes. A subsequent bone marrow biopsy reveals variably infiltrated small mature lymphocytes. Philadelphia chromosome is negative. Based on the history/physical examination and laboratory/diagnostic studies, what is the most likely diagnosis? 1 Hairy cell leukemia (HCL) 2 Acute myeloid leukemia (AML) 3 Acute lymphoblastic leukemia (ALL) 4 Chronic lymphocytic leukemia (CLL) 5 Chronic myeloid leukemia (CML)

4 Chronic lymphocytic leukemia (CLL)

A 46-year-old female patient underwent elective cholecystectomy. The attending nurse noted mild bleeding at the site of IV line and the incision site during dressing. The patient also complained of bleeding from the gums and nose. Coagulation profiles revealed prolongation of aPTT, PT, and TT along with decreased fibrinogen level and increased levels of fibrinogen degradation product (FDP). Platelet count was also decreased. The patient was not suffering from any bleeding disorder previous to her hospitalization. What is the most likely cause of this patient's bleeding tendency? 1 Deficiency of clotting factors of the intrinsic pathway 2 Deficiency of clotting factors of the extrinsic pathway 3 Platelet functional defect 4 Disseminated intravascular coagulation (DIC) 5 Cholestatic jaundice

4 Disseminated intravascular coagulation (DIC)

A 66-year-old man presents with a 2-month history of fatigue. He reports that he has recently joined Alcoholics Anonymous. On examination, he is malnourished and pale, but his neurological examination is essentially normal. A peripheral blood smear reveals macrocytic red cells. What is the most likely diagnosis? 1 Iodine deficiency 2 Zinc deficiency 3 Copper deficiency 4 Folic acid deficiency 5 Cobalamin deficiency

4 Folic acid deficiency

A 17-year-old African-American woman with known sickle-cell disease presents with right hip and chest pain. She has frequent pain crises and is well known to the sickle cell service at your hospital due to her frequent admissions and problems with narcotics abuse in the past. She says that she has had bronchitis the past few days and believes that this is what brought on this pain crisis. She says that her regimen of oral ibuprofen and hydrocodone has not relieved her pain, and she is requesting IM meperidine (Demerol). On exam she appears uncomfortable. She has a temperature of 100.7F, pulse of 99, respiratory rate of 24, and blood pressure of 105/77. Auscultation of her lungs reveals bibasilar rales, and she has a II/VI systolic murmur. Her abdomen is soft and nontender; no spleen is felt. She has pain on palpation and movement of her right hip. Laboratory evaluation reveals a white blood cell count of 15,000/mm3, with a hemoglobin of 6.8 g/dL; serum LDH is pending. What should you do next? 1 Give her prescriptions for hydrocodone and antibiotics and have her follow up with her hematologist tomorrow 2 Give her an injection of meperidine (Demerol), prescribe antibiotics for bronchitis, and have her follow up with her hematologist tomorrow 3 Give her an injection of saline to see if this is narcotic seeking behavior 4 Get a STAT chest X-ray to assess for acute chest syndrome 5 Admit to the general medical service for IV hydration and narcotics

4 Get a STAT chest X-ray to assess for acute chest syndrome

A 17-year-old boy presents for a follow up regarding fatigue and dizziness. After his initial presentation, he had some general blood work drawn, including a complete blood count (CBC). The results indicate that the patient is anemic, so further workup needs to be done in order to characterize the type and cause of his anemia. What detail from the patient's history would indicate that he may have iron deficiency anemia specifically? 1 He has been having heart palpitations. 2 He has a history of lymphoma. 3 He has been on doxycycline for 6 months. 4 He has had a craving to eat ice. 5 He also has systemic lupus erythematosus.

4 He has had a craving to eat ice.

A 34-year-old woman presents with fatigue, weakness, shortness of breath, nosebleeds, bruising, and abdominal pain. She denies recent illness. She is taking oral hypoglycemic medications daily. On physical exam, her skin is pale; you note abdominal tenderness, an enlarged spleen, sternal tenderness, and an irregular, rapid heart beat. What is the most likely diagnosis? 1 Iron deficiency anemia 2 Folic acid deficiency anemia 3 Pernicious anemia 4 Hemolytic anemia 5 Sickle cell anemia

4 Hemolytic anemia

A mother presents with her 5-year-old Caucasian son; he has a significantly swollen right knee. She states that her son has a blood coagulation disorder and frequently bleeds into his joints when he sustains any injury. His past medical history includes the use of the blood product cryoprecipitate or factor VIII concentrates for treatment for his disorder. From what disorder does this patient suffer? 1 A coagulation disorder due to liver disease 2 Hemophilia B (Christmas disease) 3 von Willebrand's disease 4 Hemophilia A 5 DIC with thrombosis

4 Hemophilia A

A 65-year-old woman presents after being on hemodialysis for the past 1.5 years for diabetic nephropathy. Despite stable subcutaneous erythropoietin and intravenous iron doses, her hemoglobin level has decreased from 12 g/dL to 10 g/dL over the past month. Oral lanthanum doses have been stabilized. Diabetes is treated with glyburide. She developed a foot ulcer 2 weeks ago and was treated with antibiotics. Additional tests include: Ferritin 800 ng/dL Transferrin Saturation 20% Total white blood cell count 7x103/mm3 Platelet count 250,000/mL TSH (thyroid stimulating hormone) 0.5 ng/mL PTH (Parathryoid hormone) 160 pg/mL (150-300 pg/mL normal for dialysis patients, National Kidney Foundation) Aluminum level, serum Normal What is the most likely cause of her anemia? 1 Iron deficiency 2 Pernicious anemia 3 Myelodysplasia 4 Inflammation 5 Hyperparathyroidism

4 Inflammation

A 90-year-old woman of Swedish ethnic background has been living in a nursing home for 7 years. She has severe megaloblastic anemia, sensory ataxia, and circulating antibodies to gastric intrinsic factor. The likely diagnosis is pernicious anemia. What dietary supplements would most likely ameliorate her symptoms? 1 Dietary supplements of vitamin C 2 Perenteral administration of vitamin D 3 Dietary supplements of B6 4 Perenteral administration of vitamin B12 5 Addition of iodized salt to her diet

4 Perenteral administration of vitamin B12

The mother of a 3-year-old boy asks to have a blood test done on her son for lead poisoning. He has not been tested before. They have moved into an older home built before 1960. She has noticed some peeling paint on windowsills and doors and has seen small paint chips on the floors. They are now having the house repainted and are staying with relatives. A careful environmental history is obtained, risk reduction and nutrition education is provided. His fingerstick blood lead level comes back at 13mcg/dL. Which additional management should be done at this level? 1 Collaborate with lead poisoning preventive program to provide home inspection and other services 2 Repeat with a venous sample within 1-3 months 3 Obtain abdominal radiographs and order bowel decontamination if indicated 4 Repeat capillary samples, confirming with venous sample within 1 month 5 Consider retesting within 3 months

4 Repeat capillary samples, confirming with venous sample within 1 month

A 6-year-old boy is brought in after cutting his right hand 30 minutes ago. Despite continuous compression, the cut continues to ooze blood. His past medical history includes idiopathic thrombocytopenic purpura starting 8 months ago, which responded initially to intravenous immunoglobulin and resolved completely after a laparoscopic splenectomy 6 months ago. His immunizations are up to date, and he has no other relevant past medical history. Family history is also unremarkable. Vital signs are BP 110/ 80 mmHg, HR 98 bpm, RR 18 rpm, temperature 36.3° C (97.2° F). On physical examination, he is alert and oriented, showing no signs of acute distress. There are several bruises and ecchymoses around his ankles, elbows, and conjunctivae. The remainder of the physical examination shows no abnormalities. A complete blood count shows Hemoglobin 14 g/dL, Hematocrit 40%, Mean Corpuscular Volume (MCV) 88 fl, Mean Corpuscular Hemoglobin Concentration (MCHC) 29 pg/mL, leukocytes 8,000 /mm3 w/normal differential, and platelets 18,000 /mm3. A peripheral blood smear shows no abnormalities except for a paucity of platelets. What modality is most likely to yield a diagnosis? 1 Bone marrow biopsy 2 Computed tomography of the abdomen 3 Ultrasound of the left hypochondrium 4 Scintigraphy with 99mTc-labeled heat-damaged erythrocytes 5 Laparoscopy

4 Scintigraphy with 99mTc-labeled heat-damaged erythrocytes

A 12-month-old African-American boy presents with a 2-day history of irritability and refusal to bear weight. His 16-year-old mother denies any recent history of fever, vomiting, diarrhea, rash, or trauma. She does admit to giving her son a daily medicine that the pediatrician said would 'keep him from getting sick'. Family history is significant for a maternal uncle who had a stroke and died when he was 35 years of age. On exam, vital signs are stable. Patient is a well-developed, well-nourished toddler who appears irritable, but consolable. He is in no apparent distress; however, he does refuse to bear weight or play. The only significant findings on his exam are swollen hands and feet. CBC reveals WBC 18,000 mm3, with 40% neutrophils, 30% lymphocytes, and 1% monocytes. Hb is 8 g/dL and platelets are 400,000 mm3. What test would be helpful in making a diagnosis? 1 Urine analysis 2 Echocardiogram 3 CXR (AP/lateral) 4 Sickle cell solubility test 5 Blood culture

4 Sickle cell solubility test

A 22-year-old female, G1, P0, comes to the clinic for follow up. She is in her second trimester of pregnancy. She complains of fever and lethargy. On physical examination, multiple purpuric spots are identified on her skin. Additionally, the patient is noted to be pale. Peripheral blood smear review shows marked anisopoikilocytosis, fragmented red cells, and occasionally nucleated red cells. White blood cells are normal in morphology. There is thrombocytopenia. Her CBC and Coagulation results are as follows: RBC 3.5X106/ml Hemoglobin 9.5mg/dl WBC 5.1x 103/ml Platelets 75x103/ml Bleeding time 12 minutes PT 11 seconds PTT 29 seconds Fibrinogen 250mg/dl The peripheral blood smear review shows multiple fragmented red blood cells. Coombs' test is negative. Fibrin degradation products are 40ug/ml, and D-dimers are 90ng/ml. The most likely diagnosis is: 1 Disseminated intravascular coagulation (DIC) 2 Evan's syndrome 3 Idiopathic thrombocytopenic purpura (ITP) 4 Thrombotic thrombocytopenic purpura (TTP) 5 Von-Willebrand's disease

4 Thrombotic thrombocytopenic purpura (TTP)

A 66-year-old man presents with a 2-month history of numbness in his lower limbs and a feeling of fatigue. He has been a strict vegetarian for the past 2 years. On examination, he is pale; he has an ataxic gait and diminished proprioceptive sensation in his legs. A peripheral blood smear reveals macrocytic red cells. What is the most likely diagnosis? 1 Iodine deficiency 2 Zinc deficiency 3 Copper deficiency 4 Folic acid deficiency 5 Cobalamin deficiency

5 Cobalamin deficiency

A severely anemic patient is transfused with 2 units of blood. After the start of the transfusion, the patient complains of nausea and develops fever and chills. What is the mechanism of this reaction? 1 IgA-mediated 2 Exotoxin-mediated reaction 3 T cell-mediated effect 4 Type III hypersensitivity 5 Cytotoxic hypersensitivity

5 Cytotoxic hypersensitivity

An 18-month-old Caucasian girl presents for a routine visit. A screening blood test reveals anemia with a hemoglobin level of 7.5 g/dL. Her past medical history is notable for an unremarkable neonatal course, 2 episodes of acute otitis media beginning at 10 months of age, and hospitalization for bronchiolitis at 12 months of age. The patient is a "picky" eater, but she is given a balanced diet. She drinks 48 ounces of whole cow's milk a day, and the mother reports her daughter occasionally eats dirt. There is a history of anemia in the patient's mother during pregnancy. Except for pallor of the skin and conjunctiva, physical exam is normal. Results of a complete blood count are: WBC - 9,500/mm3 with 33% neutrophils, 58% lymphocytes, 6% monocytes and 3% eosinophils RBC - 2.7 million/mm3, hemoglobin 7.5 g/dL, hematocrit 23%, MCV 68 fL, MCHC 29 g/dL, RDW 15% Platelets 500,000/mm3 What is the best initial therapy for this patient? 1 Folic acid 1 mg/kg daily for a month 2 Folic acid 1 mg/kg daily for a year 3 Ferrous sulfate 2 mg/kg of elemental iron daily for a month 4 Ferrous sulfate 1 mg/kg of elemental iron twice a day for a month 5 Ferrous gluconate 3 mg/kg of elemental iron twice a day for 3 months

5 Ferrous gluconate 3 mg/kg of elemental iron twice a day for 3 months

A 13-year-old African-American boy presents with a 4-month history of increasing fatigue, pallor, exertional dyspnea, and depression. Past medical history is significant for an ischemic stroke due to sickle cell disease 6 years ago. Since his stroke, the boy has been receiving chronic red cell transfusion therapy in addition to subcutaneous iron chelation therapy started 3 years ago. On physical exam, the boy's pulse is 150 BPM, respirations are 20/min, blood pressure is 105/75 mm Hg, and O2 saturation by pulse oximetry is 92% on room air. In general, he appears ill; he is dyspneic and refuses to lie flat on the bed during his examination. Otherwise, he is cooperative with his exam. He is diaphoretic, with no rashes or skin abnormalities noted. His cardiovascular exam is significant for a 4/6 systolic ejection murmur and a gallop. His pulmonary exam demonstrates scattered expiratory wheezes throughout all lung fields. His liver is 4 cm below the right costal margin; no spleen is palpable. The remainder of his exam is unremarkable. You arrange for the patient to be transported to the hospital for admission. What study is most important for the initial evaluation and management of this patient? 1 Hemoglobin, liver function tests, hemoglobin electrophoresis, and echocardiogram 2 Hemoglobin, ferritin, liver function tests, and chest X-ray (AP/lateral) 3 Hemoglobin, arterial blood gas, hemoglobin electrophoresis, and liver biopsy 4 Hemoglobin, ferritin, chest X-ray CXR (AP/lateral), and echocardiogram 5 Hemoglobin, ferritin, liver function tests, chest X-ray, and echocardiogram

5 Hemoglobin, ferritin, liver function tests, chest X-ray, and echocardiogram

A 12-year-old Jewish girl is brought in by her father. The father reports that the child started to bleed profusely following tooth extraction 2 hours ago. Further questioning reveals that there is a history of bleeding disorder in the child's maternal uncle and aunt. Examination reveals tachycardia and bleeding from the site of tooth extraction. Coagulation profile reveals prolonged activated partial thromboplastin time (aPTT), normal prothrombin time (PT) and thrombin time (TT), and deficiency of factor XI. The child was treated with fresh frozen plasma (FFP) and fibrin glue. What is the likely diagnosis? 1 Thrombocytopenia 2 Hemophilia B 3 Von Willebrand's disease 4 Hemophilia A 5 Hemophilia C

5 Hemophilia C

A 59-year-old man starts to notice that he no longer has the energy for his morning jog. He starts to develop back pain, so he completely stops exercising; however, this does not seem to help. He is always tired and seems to have a lot of aches and pains. One day, his back pain is really bad, so he sees his primary care physician. He also acknowledges constipation and polyuria. On physical exam, his doctor notes that he is pale. His laboratory tests are as follows: TEST RESULTS REFERENCE RANGE RBC 4.2 x 106/µl 4.5 - 5.7 x 106/µl (male) 3.9 - 5.0 x 106/µl (female) hematocrit 38 40 - 50 (male) 36 - 44 (female) hemoglobin 13 gm/dL 13.8 - 17.2 gm/dL (male) 12.1 - 15.1 gm/dL (female) calcium 12 mg/100 mL 9 - 10.5 mg/100 mL ESR 40 mm/hr <30/mm hr Comment: Rouleaux formation is seen An x-ray of the lumbar spine reveals "punched out" lesions. What immunoglobulin is most frequently elevated in this condition? 1 IgA 2 IgM 3 IgE 4 IgD 5 IgG

5 IgG

A 63-year-old woman presents for her yearly exam. She has been feeling well and does not have any current complaints. She has past medical history of hypertension and hyperlipidemia. Routine laboratory testing reveals leukocytosis with 10% blasts. There is mild anemia, and platelets are normal. Peripheral blood cytometry confirms the diagnosis of chronic lympocytic leukemia. What is the appropriate treatment at this time? 1 Imatinib 2 Lenalidomide 3 Prednisone 4 Stem cell transplant 5 Observation

5 Observation

A 10-month-old African-American infant presents with a recent onset of symptoms suggestive of sickle cell anemia. He is the younger of 2 children, and his mother is concerned about his health because of the symptoms and a family history of sickle cell anemia. What are the 1st symptoms usually seen in these children? 1 Retinal deterioration 2 Yellowing of skin and eyes 3 Delayed growth 4 Tachycardia and fever 5 Painful swelling of hands and feet

5 Painful swelling of hands and feet

A 40-year-old woman, undergoing a gynecologic workup for metromenorrhagia, presents with a several-week history of fatigue and lightheadedness. Laboratory evaluation reveals a hemoglobin level of 11 g/dL (12 - 16 g/dL), hematocrit of 34%, MCV of 70 fL (80 - 100 fL), and MCH of 24 pg (27 - 33 pg). Further studies reveal a ferritin level of 25 ng/mL (12 - 300 ng/mL), TIBC of 500 mcg/dL (250 - 450 mcg/dL), and an iron level of 45 mcg/dL (60 - 170 mcg/dL). The patient is diagnosed with iron-deficiency anemia secondary to menstrual blood loss, and she is given a prescription for ferrous sulfate 325 mg po tid and told to follow up in 1 month. The patient presents to the emergency department several days later. Her husband states that she had a sudden onset of dyspnea at rest and chest pain earlier that evening. The patient is extremely pale and notes continued chest pain and lightheadedness. Her EKG is unremarkable and lungs are clear to auscultation. Oxygen and nitroglycerin are administered and cardiac enzymes are drawn. The patient's hematocrit is now 23% with a hemoglobin level of 7.5 What treatment is most appropriate for this patient at this time? 1 Increasing the dose of ferrous sulfate 2 Changing the oral medication to a combination of ferrous fumarate, ascorbic acid, folic acid, and cyanocobalamin 3 Iron dextran IM or IV in the emergency department 4 Iron sucrose 200 mg IV in the emergency department with 4 more 200 mg IV doses to be given over the next 14 days 5 Red blood cell transfusion

5 Red blood cell transfusion

A 30-year-old African-American man presents for screening for an abnormal hemoglobin carrier state. His vital signs show a pulse of 90 BPM, blood pressure of 120/75 mm Hg, and temperature of 98.4° F. His physical examination is unremarkable. A CBC is done as part of his premarital work-up and the results are as follows: Test Result Normal Value RBCs 5.0 x 106/mL 4.5 - 5.5 x 106/mL Hemoglobin 13.0 g/dL 13.5 - 16.5 g/dL HCT 41% 41 - 50% MCV 82 fL 80 - 100 fL MCH 26 pg 26 - 34 pg MCHC 32% 31 - 37% WBCs 7.0 x 103/mL 4.5 - 10 x 103/mL Platelets 170 x 103/mL 100 - 450 x 103/mL Morphologic review of the peripheral blood smear shows normal erythroid, platelet, and white blood cell morphology. What is the best management at this time for this patient? 1 No therapy necessary for the patient, but his fiancé also needs to undergo Hb electrophoresis 2 The patient needs to be treated 3 The patient needs to make certain life adjustments, but no therapy is necessary 4 The patient needs to pursue genetic counseling only if the hemoglobin S quantitation exceeds 50% of the total hemoglobin 5 The patient needs to be evaluated by Hb electrophoresis

5 The patient needs to be evaluated by Hb electrophoresis

A 29-year-old woman presents for a general physical examination. Her vital signs are pulse 85/min, BP 120/80, and temperature 98.9°. Her physical examination is unremarkable. The patient is a vegetarian. Her CBC and red cell indices are as follows: RBCs 3.5 x 106/mL (3.5 - 5.5) Hemoglobin 10.5 g/dL (11.0 - 16.0) HCT 35% (40% - 54%) MCV 101 fl (80 - 100 fl) MCH 30 pg (27 - 33 pg) MCHC 30 g/dL (32 - 36 g/dL) WBCs 9.0 x 103/mL (3.8 - 10.8 x 103/mL Platelets 210 x 103/mL (150 - 400) Serum B12 and folate are performed. The results of the 2 tests are as follows: Serum B12 95 pg/mL Serum folate 12 ng/mL Red cell folate 150 ng/mL What is the cause of this patient's megaloblastic anemia? 1 Cannot be determined from these studies 2 Combined folate and vitamin B12 deficiency 3 Folate deficiency 4 Myelodysplastic syndrome 5 Vitamin B12 deficiency

5 Vitamin B12 deficiency

You are following up on laboratory results for your supervising physician while she is out of town. A bone marrow biopsy result for a 62-year-old man is noted. Bone Marrow aspiration (biopsy) Infiltration with small lymphocytes, B-cell. Coexpression of CD19, CD20 and CD5 On further chart review, you see that this patient presented to your clinic for review of abnormal health fair screening tests for his employment. The patient reported that he felt well in general, but he added that he could not exercise as vigorously as he could in the past; however, he attributed that to his aging. The table shows his complete blood count. WBC 98.4 Neutrophils 5.7% Lymphocytes 91% If you were to review this patient's physical exam from his prior visit, what finding would you expect to see in his chart? 1 Ecchymoses 2 Exudative pharyngitis 3 Fever 4 Jaundice 5 Large mass 6 Lymphadenopathy 7 Petechiae

6 Lymphadenopathy

You are following up on laboratory results for your supervising physician while she is out of town. A bone marrow biopsy result for a 62-year-old man is noted. Bone Marrow aspiration (biopsy) Infiltration with small lymphocytes, B-cell. Coexpression of CD19, CD20 and CD5 On further chart review, you see that this patient presented to your clinic for review of abnormal health fair screening tests for his employment. The patient reported that he felt well in general, but he added that he could not exercise as vigorously as he could in the past; however, he attributed that to his aging. The table shows his complete blood count. Once this patient has been referred for treatment and achieves disease remission, he returns to his primary care clinic for health maintenance. For what complication, which is specifically related to his condition, should the primary care physician monitor? 1 A second malignant tumor 2 Decreased visual acuity 3 Hypercoagulability 4 Osteoporosis 5 Valvular heart disease

1 A second malignant tumor

A 32-year-old man experiences repeated heavy exposure to benzene while working in a petroleum refinery. He develops aplastic anemia and requires an allogenic bone marrow transplant. 10 days after the bone marrow transplant, the patient develops a cytomegalovirus pneumonia, severe skin rash, and jaundice. What is the most likely diagnosis? 1 Acute graft-versus-host (GVH) disease 2 Chronic GVH disease 3 Transplant rejection 4 Goodpasture syndrome 5 Hashimoto disease

1 Acute graft-versus-host (GVH) disease

A 16-year-old African-American girl with Hemoglobin S disease has experienced numerous complications of her condition, including multiple acute splenic sequestration crises that resulted in a splenectomy when she was 14 years old. She is no longer receiving antibiotic prophylaxis. Her last polyvalent pneumococcal vaccine was 2 years ago. She presents to the emergency department with a 2-day history of fever as high as 104.3°F, without localizing symptoms. She notes malaise, abdominal discomfort, and 1 episode of vomiting. Examination reveals a temperature of 103.2°F; pulse 110; blood pressure 116/76 mm Hg; and respirations 21. Hydration is normal, and no localizing findings are present. What is the next step in the management of this patient? 1 Admit the patient, obtain cultures, and immediately begin IV ceftriaxone and vancomycin 2 Await input from the infectious disease consultant 3 Discharge the patient on oral amoxicillin/clavulanate 4 Examine a peripheral blood smear for Howell-Jolly bodies 5 Obtain blood cultures and discharge the patient pending results

1 Admit the patient, obtain cultures, and immediately begin IV ceftriaxone and vancomycin

A 70-year-old man presents for a follow up with his cardiologist. There are no specific complaints. Findings at the physical exam are BP 130/80 mmHg, HR 80 beats/min, and the appearance of pale mucous membranes. Lungs are clear to auscultation, and there is no edema of lower extremities. Fecal occult blood test (FOBT) was negative. Blood test shows hypochromic microcytic RBCs. Further exams show low serum iron, low total iron-binding capacity (TIBC), and increased ferritin. What is the most likely diagnosis? 1 Anemia of chronic disease 2 Anemia secondary to iron deficiency 3 Beta thalassemia 4 Megaloblastic anemia 5 Sideroblastic anemia

1 Anemia of chronic disease

A 24-year-old woman is evaluated in the endocrinology clinic. She is on 100 mcg of thyroxine per day for treatment of Hashimotos's thyroiditis and receives oral steroids for autoimmune adrenal insufficiency. In addition, she complains of fatigue, numbness and tingling in her feet. When routine labs were drawn, the following values were noted: Hb: 9.9 g/dl MCV: 102 Iron indices: normal Folate: normal B12: low Parietal cell antibodies: high What would prevent the development of megaloblastic anemia in this woman? 1 Intramuscular injection of B12 2 Intravenous folate 3 Prescribed dose of oral folate 4 Small dose of oral B12 5 Prescribed dose of oral ferrous sulphate

1 Intramuscular injection of B12

A 64-year-old man presents to his GP for the first time in 4 years with a recent history of fatigue, pallor and worsening back pain in the lumbar region for the past 3-4 months. He denies any injury to his back or similar pain in the past. He had a colonoscopy 3 months prior, where 2 benign polyps were removed without further abnormalities. At that time, he also had a PSA level of 3.1 ng/ml. On examination, BP: 143/92 mm Hg, P: 75 bpm and regular, Resp: 16/minute, Temp: 97.44°F. Skin exam reveals marked pallor without lesions or bruising. Musculoskeletal examination reveals bony tenderness along lumbar vertebrae without paraspinal muscle tenderness to palpation. Decreased range of motion of spine noted on examination due to pain elicited during examination. The remainder of the examination including digital rectal exam, focal neurological, pulmonary, and cardiac exams are within normal limits. Which of the following tests will be most helpful in making your diagnosis? 1 Serum protein electrophoresis 2 Complete blood count with differential and platelets 3 Serum calcium level adjusted for serum protein 4 Total serum protein 5 Urine albumin

1 Serum protein electrophoresis

A 38-year-old female gave birth to a healthy baby girl 3 months ago. Her pregnancy and vaginal delivery were unremarkable. Over the past 3 months, she developed increased oral bleeding with hemorrhagic bullae, and you suspect Idiopathic Thrombocytopenic Purpura (ITP). Which of the following would make the diagnosis of ITP less likely? 1 Splenomegaly 2 Petechiae 3 Megakaryocytosis 4 Increased bruising 5 Epistaxis

1 Splenomegaly

A 5-year-old girl is brought to her pediatrician by her mother. Her mother notes that t he patient has been bruising easily for the last few weeks, but she does not recall any major injury. The patient began feeling tired around this time as well. The mother initially thought she had the flu, but she brought the patient in when it became apparent she was not improving. On examination, the patient is pale and appears fatigued. Her skin has multiple areas of bruising and petechiae. She is febrile. A CBC/Diff is significant for the following: WBC 72.2 x 103mm3 Hgb 8.4 gm/dl Hct 22.3% Platelets 30 x 103/µL Neutrophils 5% Lymphocytes 0% Monocytes 0% Basophils 0% Eosinophils 0% Blasts 95% Bone marrow biopsy is obtained and confirms presence of blasts. Stains for myeloperoxidase and Sudan black are negative. What is the most likely diagnosis? 1 Acute myeloid leukemia 2 Acute lymphocytic leukemia 3 Chronic myeloid leukemia 4 Chronic lymphocytic leukemia 5 Hairy cell leukemia

2 Acute lymphocytic leukemia

A 35-year-old man with known stage 3 chronic kidney disease due to diabetes presents with fatigue. His blood sugars are acceptable; his mood is euthymic, and he is sleeping and eating well. He does not smoke, and he has no known toxic environmental exposures. Work up shows normochromic normocytic anemia, with a hemoglobin of 11 g/dl, which is decreased from 12 g/dl 6 months ago. Medications include lisinopril, furosemide, and insulin. Iron 50 ug/dl (normal 50-150 ug/dl) Ferritin 200 ng/ml Ratio soluble transferrin receptor/log ferritin Reduced Reticulocytes 0.25% Platelets Normal Total white blood cell count 5x103/mm3 B12 levels, serum Normal What is the most likely diagnosis? 1 Iron deficiency anemia 2 Anemia of chronic disease 3 Thalassemia 4 Myelodysplasia 5 Pernicious anemia

2 Anemia of chronic disease

You are following up on laboratory results for your supervising physician while she is out of town. A bone marrow biopsy result for a 62-year-old man is noted. Bone Marrow aspiration (biopsy) Infiltration with small lymphocytes, B-cell. Coexpression of CD19, CD20, and CD5 In further chart review, you see that this patient presented to your clinic for review of abnormal health fair screening tests for his employment. The patient reported that he felt well in general, but he added that he could not exercise as vigorously as he could in the past; however, he attributed that to his aging. The table shows his complete blood count. What pharmaceutical treatment should primarily be used to treat his condition, especially as it progresses? 1 Anticoagulants 2 Chemoimmunotherapy 3 Corticosteroids 4 Disease-modifying anti-rheumatic drugs (DMARDs) 5 Tumor necrosis factor (TNF) inhibitors

2 Chemoimmunotherapy

A 35-year-old woman underwent elective cholecystectomy for obstructive jaundice and was given heparin prophylaxis for postoperative thromboembolism. The patient has started to bleed at the incision site, and her partial thromboplastin time (PTT) is prolonged. What is a possible cause of her bleeding? 1 Vitamin K deficiency 2 Heparin therapy 3 Congenital deficiency of factor VII 4 Afibrinogenemia 5 Disseminated intravascular coagulation (DIC)

2 Heparin therapy

A 35-year-old man with stage 3 chronic kidney disease has had a decreasing hemoglobin count over the past year. He is asymptomatic. Work up shows: Hemoglobin 10.5 g/dl Iron 49 ug/dl (normal 50-150 ug/dl) Transferrin Saturation 18% Ferritin 150 ng/ml Stool occult blood Negative, 3 samples B12 Normal Folate Normal TSH Normal Reticulocytes Low What treatment is suggested? 1 Whole blood tranfusion 2 Oral iron supplements 3 Intravenous iron supplements 4 Erythropoietin injections 5 Erythropoietin+iron supplement

2 Oral iron supplements

A 16-year-old girl presents for the third time in 2 months with symptoms of an upper respiratory tract infection. She has a blocked nose, tearing eyes, and appears exhausted. The first 2 times, she presented the same way and the diagnosis was a viral upper respiratory infection. At the 2nd presentation, she was given antibiotics in the vain hope that there was a superimposed sinus bacterial infection. When she presents the 3rd time, her mother comes in with her and demands to know what is making her daughter so susceptible to viral infections. The patient is a high school student and has a heavy study load. 2 months ago, she was studying for finals, and everyone believed the stress of exams was the cause of her illness. This time, she has been sick for 3 days with a bad cough, fever, runny nose, and throat pain; she denies that she has been overworked and says that she eats a good diet. On exam, you observe temperature 100.9°F; pulse 92 bpm; RR 18/min; BP 120/80 mm Hg. The head/ear/throat exam is unrevealing; the lung and heart exam is within normal limits. Her laboratory data reveals: CBC Normal Ranges Hb - 12 gm/dl (Normal: 12-16 g/dL) Hct - 36 % (Normal: 37-47) MCH - 30 pg/cell (Normal: 26-34) MCV - 80 fl (Normal: 80-96) MCHC - 33% (Normal: 32-36) WBC - 10,000/mm3 (Normal: 4500-11,000) NS - 75% Lymph - 20% Baso - 2% Mono - 2% Eos - 1% Chem 7: (Na/K/Cl/Co2/Cr/BUN/BS) within normal. What test should be ordered for the patient? 1 Throat culture 2 Serum Iron test 3 Serum Immunoglobulin levels 4 Folate level 5 No test is indicated at this time

2 Serum Iron test

A 38-year-old woman was treated for chronic myelogenous leukemia, with an allogeneic bone marrow transplant. She was released from the hospital with prescriptions for cyclosporine and steroids. She was readmitted 5 weeks after the transplant, noting abdominal cramping, nausea, and watery diarrhea. Over the last 2 weeks, she had developed a skin rash on her hands and feet, and the rash had spread to other parts of her body, peeling and blistering, giving her skin the appearance of severe sunburn. Physical examination revealed mild jaundice. Bloodwork showed elevated levels of bilirubin, SGOT, and alkaline phosphatase. What is the most likely diagnosis? 1 Hepatitis B 2 Chronic Graft vs Host disease (GVHD) 3 Acute Graft vs Host disease (GVHD) 4 HIV 5 Relapse of chronic myelogenous leukemia (CML)

3 Acute Graft vs Host disease (GVHD)

Screening test for platelet dysfunction is 1 Prothrombin time 2 Activated partial thromboplastin time 3 Thrombin time 4 Bleeding time

4 Bleeding time

A 60-year-old man presents with sharp pain in his ribs and sternum; he has a history of allergies and has been sneezing frequently. Except for a heart rate of 120/min, the physical examination is normal. With the exception of a tonsillectomy as a child, his medical and surgical history is unremarkable. He has chronic seasonal allergies. Family history reveals nothing significant. Radiographs on the ribs and sternum reveal lytic lesions and a significant fracture of the sternum. Neurological examination is unremarkable. The laboratory results are shown in the table. The following cells are occasionally seen in the peripheral stained blood smear, in addition to rouleaux formation (see figure1). Bone marrow aspiration is performed Tests Results BUN 24 mg/dL Protein 12.0 g/dL Albumin 3.8 g/dL Uric Acid 11.9 mg/dL Calcium 8.4 mEq/L Sodium 137 mmol/L Urinalysis Protein 4+ RBC 4.3 x 1012/L Hgb 12.4 g/dL Hct 0.35 MCV 82 fL MCHC 34% MCH 28 pg WBC 8.0 x 109/L Differential Metas 0.3% Bands 3.2% Segs 55.3% Eos 4.0% Basos 0% Lymphs 34.0% Monos 3.2% Platelets 380 x 109/L Based on the clinical presentation and laboratory test results, what is the most likely diagnosis? 1 Burkitt lymphoma 2 Acute lymphocytic leukemia 3 Multiple myeloma 4 Acute myeloblastic leukemia 5 Chronic lymphocytic leukemia

3 Multiple myeloma

A 68-year-old woman presents with a 2 - 3 month history of dyspnea on exertion, easy fatigability, and lassitude. She states that recently she is having difficulty swallowing. She denies hemoptysis, GI, or vaginal bleeding. She claims her diet is good, but her appetite has varied. On physical examination, other than pallor, no significant physical findings are noted. Occult blood is negative. A hemoglobin and hematocrit are ordered; the results are 7.2 g/dL and 22% respectively. She is diagnosed with iron-deficiency anemia. The results of a barium esophagram reveal esophageal webbing. What is the most likely diagnosis? 1 Barrett's esophagus 2 Achalasia 3 Plummer-Vinson syndrome 4 Dressler's syndrome 5 Mallory-Weiss syndrome

3 Plummer-Vinson syndrome

A 17-year-old girl presents to the clinic due to chronic fatigue. Her past medical history is unremarkable. There is no history of surgeries. Review of systems reveals heavy monthly menses since menarche at age 13. She admits to using more than 20 tampons per day on the heaviest 3 days of her menstrual cycle. Her last menstrual cycle commenced 1 week ago. There are no recent medications. Exam is within normal limits, with no vaginal discharge. Labs are as follows: pregnancy test negative; cervical cultures negative; thyroid studies within normal limits; Hb 10g/dL, WBC 9,000/μL, platelet count 250 x 10³/μL. Peripheral blood smear is consistent with microcytic, hypochromic anemia. Prothrombin time, partial thromboplastin time (PT, PTT), and fibrinogen are normal. Bleeding time is prolonged. What is the most likely diagnosis? 1 Endometriosis 2 Ectopic pregnancy 3 Von Willebrand disease 4 DIC 5 Hemophilia A

3 Von Willebrand disease

A 29-year-old African-American man develops dysuria and increased frequency of micturition. In the emergency room, he is found to have a urinary tract infection and is treated with trimethoprim-sulfamethoxazole (Bactrim), as well as recommended to follow up with his primary care physician in the office. Over the next few days, the patient experiences fatigue, fever, jaundice, abdominal and back pain, and dark urine. Blood tests show Hb of 4g/dl, reticulocyte count of 6%, and MCV of 93. Peripheral smear reveals cell fragments, microspherocytes, and blister or bite cells. Heinz bodies are also present. Based on these findings, which clinical entity is most likely? 1 Hereditary spherocytosis 2 Sickle cell anemia 3 Porphyria 4 G6PD deficiency 5 Autoimmune hemolytic anemia

4 G6PD deficiency

A 58-year-old male presents to the office for routine follow-up of hypertension. A CBC on the front of the chart reveals: WBC 7.0 x 103/mm3 4.5 - 11 RBC 6.5 x 106/μl 4.6 - 6.2 (male) Hematocrit 58 % 40 - 54 (male) MCV 68 μ3 80 - 100 RDW 14.9 % 11.6 - 14.6 Platelets 250,000 mm3 150,000 - 450,000 Upon questioning, he admits that he has not been compliant with diet or exercise and continues to smoke. On examination height is 5'8, weight is 210 lbs, with a BMI of 32. Blood pressure is 156/88 mm Hg, pulse is 88/min, temperature 98.6°F, and respirations 16/minute. Skin is dry without rubor or cyanosis. On auscultation, lungs are clear and the heart has a regular rhythm without murmur, gallop, or rub. Abdominal exam reveals truncal obesity, active bowel sounds, and no organomegaly. The remainder of the PE is normal. Office pulse oximetry reveals a saturation of 92%. You send a blood specimen for an erythropoietin level. In the event the erythropoietin level is normal, the most likely diagnosis is 1 Polycythemia vera 2 Hydronephrosis 3 Sleep apnea 4 Gaisböck's syndrome 5 Carboxyhemoglobinemia

4 Gaisböck's syndrome

A 43-year-old woman presents with fever and cough, and she is found to have right lobar pneumonia with mild pleural effusion. She is admitted to the hospital for IV antibiotics and hydration. Not much of a medical history is obtained because she is quite confused on the first day and no family members are present. Based on her partial list of medications, it seems she has hypertension, systemic lupus erythematosus, and arthritis. Surgical history, family history, and allergies are unknown since this is her first visit to this hospital. 2 IV lines are started: 1 in each arm, 1 for fluids, and the 2nd for the 2 antibiotics. Other orders include nebulizer treatment every 4 hours, oxygen by nasal cannula, CXR in the morning, hep-lock second IV in between antibiotics, and a Foley catheter for strict urine output measurement; the catheter shows amber colored urine on admission. The patient improves gradually. By the 2nd day, she is fully alert, oriented, and far less symptomatic. She is afebrile, but she still has a mild productive cough and occasional shortness of breath. On the same day, she also complains of left arm pain and has swelling of the entire arm with much tenderness diffusely, indicating possible upper extremity thrombosis. She is given a bolus of heparin and a heparin infusion is initiated. Five days later, her labs show a Hb of 12g/dl, WBC 11,000, down from 17,000, and platelet count of 56,000, down from 250,000 on admission. Her EKG is normal sinus rhythm, troponin is 0.1, CK 67 and CXR show decreased consolidation with resolving of pleural effusion. What is the most likely diagnosis? 1 Lupus anticoagulant 2 Oral contraceptive induced thrombosis 3 Trousseau's syndrome 4 Heparin-induced thrombocytopenia 5 Paroxysmal nocturnal hemoglobinuria (PNH)

4 Heparin-induced thrombocytopenia

A 6-year-old boy is brought to the emergency room with chest pain and signs of respiratory distress. He has had a minor upper respiratory tract infection that suddenly worsened with temperature elevation. Physical examination finds increased pulmonic second heart sound and systolic ejection murmur. Chest x-ray shows lobar infiltrates. Leukocyte count is 18,000/mm3, hemoglobin 7g/dl, and reticulocytes 12%. What is the most likely cause of chest pain and respiratory distress? 1 Splenic sequestration 2 Myocardial infarction 3 Mycoplasma pneumonia 4 Sickle cell disease 5 Pneumocystis carinii infection

4 Sickle cell disease

A 3-day-old baby boy starts to have mild epistaxis after vaginal delivery at home. Mom is a 38-year-old G2P2 who had diet-controlled gestational diabetes. She took a prenatal vitamin and iron supplement. Active labor lasted 3 hours. The midwife who examined the baby after the delivery declared the baby healthy. Since the delivery, the mother has been exclusively breastfeeding. The baby is eating every 1-2 hours and has had several wet diapers and 3 stools. What treatment should be administered for the epistaxis? 1 Fresh frozen plasma 2 Whole blood transfusion 3 Pressure to nares 4 Vitamin K infusion 5 Chemocautery

4 Vitamin K infusion

A 28-year-old female administrator is referred to your office for further evaluation. She has noted swellings on her neck for the past 3 weeks. She denies any history of cough, fever, or night sweats. She reports she has been getting pruritus after taking alcohol. She is not on any medication and has no prior admissions. On examination, she is not pale and her vital signs are normal. She has a 2cm rubbery, non-tender left anterior cervical node and a 3cm left posterior cervical node. No other nodes are palpable. The systemic examination is essentially normal. CT scans done of the chest, abdomen and pelvis are normal. Reed-Sternberg cells are seen on a biopsy of the node. The ESR is 30 mm/hr. What is the appropriate therapy for this patient? 1 Radiotherapy only 2 Chemotherapy only 3 Surgical resection 4 Broad-spectrum antibiotics 5 Radiotherapy and chemotherapy

5 Radiotherapy and chemotherapy

A 45-year-old Caucasian man is employed to demolish numerous painted structural steel railroad trestles. The project is scheduled to take 2 years. He is not given protective masks or clothing. Towards the end of the project, he starts to develop abdominal pain, so he sees his family doctor. On questioning during the medical history, he comments that he frequently drops things with his right hand. On physical examination, his lungs are clear to auscultation and no abnormal breath sounds are heard. His cardiac exam is unremarkable. His skin exam is normal other than a pigmentation at the gingivodental margin. His extensor muscles of the wrist and finger are weak. A CBC is done; the results are shown in the chart. TEST RESULTS REFERENCE RANGE RBC 4.5 x 106/µl 4.5 - 5.7 x 106/µl (male) 3.9 - 5.0 x 106/µl (female) Mean corpuscular volume 73 cu µ 80 - 100 cu µ Hematocrit 30 40 - 50 (male) 36 - 44 (female) Hemoglobin 10 gm/dL 13.8 - 17.2 gm/dL (male) 12.1 - 15.1 gm/dL (female) Platelets 250,000/mm3 130,000 - 400,000 mm3 Comments: Red blood cell stippling is present. Hypochromic red blood cells are present. What additional laboratory test would be useful? 1 Protoporphyrin 2 Ceruloplasmin 3 Ferritin 4 TIBC 5 Serum folate

Protoporphyrin

A 45-year-old man is evaluated for a 6-month history of palpitations, easy fatigability, and chest pain on exertion. He is a nonsmoker and does not consume alcohol; he has no significant past medical history. His body weight has remained stable. He has spent the last 2 years travelling the world. His wife says that he eats "unhealthily". He is a strict vegan, consumes a lot of snacks, and has never taken any vitamin supplements. Examination shows a 5'7" male with a BMI of 19. His BP is 130/70 and pulse 90/min; his temperature is 98.4 degrees Fahrenheit. Conjunctival pallor is present. Auscultation shows a grade 2/6 murmur ejection systolic murmur heard all over the precordium. Lab investigations are sent and are given below: Hb: 8.8 g/dl WBC: 6000/cubic mm MCV: 72 Ferritin: low TIBC: elevated Stool occult blood: negative ESR: 8 mm/hour TSH: normal What is the most appropriate medical therapy? 1 Oral folate with B12 2 Intramuscular iron dextran 3 Oral ferrous sulphate 4 Packed red cell transfusion 5 Erythropoietin injection

3 Oral ferrous sulphate

A 17-year-old boy presents with abdominal pain following a motor vehicle accident. The patient was the passenger in the vehicle and was wearing restraints. He states that he has pain in the left side of his abdomen, which began at the time of the accident. Physical exam revealed a well-developed young man in moderate distress. He had left upper quadrant tenderness, nausea, and positive Kehr's sign. He also had diaphoresis, tachycardia, and hypotension. What is the most likely cause of this patient's symptoms? 1 Ruptured abdominal aorta 2 Hepatic injury 3 Splenic injury 4 Ruptured diaphragm 5 Diverticulitis

3 Splenic injury

A 67-year-old man is recovering following an uncomplicated total hip arthroplasty. His past medical history is significant for hypertension and mild asthma. On postoperative day 2, he begins to note stiffness in his right leg. On exam, his right calf is noted to be swollen and slightly warmer than the left leg. The remainder of the exam is unremarkable. He is afebrile, and his vital signs are stable. What diagnostic evaluation is most appropriate in this situation? 1 Compression ultrasonography 2 Contrast venography 3 Contrast-enhanced MRI 4 Lower extremity X-ray 5 Serum B-type natriuretic peptide (BNP) measurement

1 Compression ultrasonography

A 45-year-old dialysis patient has a 4-month history of low hemoglobin levels, despite increasing her erythropoietin dose to 300 unit/kg/week. She has been on dialysis for 1 year due to hypertensive nephrosclerosis. She does not take ACE inhibitors; she has had no infections, surgeries, or transplants; age-appropriate cancer screening has been negative. Diagnostic studies include: Hemoglobin 10.5 g/dl Transferrin saturation 30% Ferritin 400 ng/ml B12, Folate, TSH Normal, normal, normal Anti Red cell antibodies normal HIV testing negative LDH normal Aluminum, serum normal What is the next step in the management of this patient? 1 Evaluate PTH levels 2 Supplement iron 3 Give chelators 4 Supplement vitamin C 5 Give carnitine

1 Evaluate PTH levels

A 36-year-old female on a visit to her primary care physician complained of easy fatigability for the last several months. She has 3 children and is a homemaker. On further questioning she denies any abdominal pain or dyspepsia like symptoms. Her menstrual history includes irregular periods every 25-40 days lasting for 8 days. Her vitals are normal and physical exam reveals mild pallor only. Her Hb was 8.2 gm%, WBC 6100/uL, platelets 350000/uL, MCV is 76 fL and MCHC is 29%. What is the next step in the diagnosis of the condition? 1 Serum iron, TIBC and ferritin 2 Bone marrow biopsy 3 Erythropoietin level 4 Pelvic ultrasound 5 Serum LDH

1 Serum iron, TIBC and ferritin

A 55-year-old Caucasian post-menopausal woman comes to your office for follow-up of her hypertension, which she has had for 10 years. Her most recent blood pressure was 130/80 mm Hg. You review her lab values and note that her serum creatinine is 1.5 mg/dL. Her hemoglobin (Hgb) is 11.5 g/dL. What is the next best step in the management of this patient? 1 Anemia work-up and treatment when her hemoglobin <11 g/dL 2 Anemia work-up and treatment when her hemoglobin is 9 g/dL 3 Anemia work-up and treatment when her hemoglobin is 8 g/dL 4 Anemia work-up and treatment now, when hemoglobin is 11.5 g/dL 5 Prescribe erythropoetin and monitor hemoglobin

4 Anemia work-up and treatment now, when hemoglobin is 11.5 g/dL

A 7-year-old African-American girl presents with a 6-hour history of severe pain in her hands and feet, which started spontaneously. She has been having bouts of excruciating pain since she was 6 months old. She is an only child, and her parents have recently discovered that they are both carriers of her condition. On examination, her HR is 115 bpm, and blood pressure is 90/50 mm Hg; she is also jaundiced. She has a mildly enlarged spleen, but no source of infection. Laboratory investigations reveal hemoglobin of 8 g/dl? What treatment would be most appropriate for this patient? 1 Splenectomy 2 Ferrous sulphate 3 Blood transfusion 4 Bone marrow transplant 5 Intravenous hydration

5 Intravenous hydration

A 54-year-old male presents to the office for routine follow-up of hypertension. A CBC on the front of the chart reveals: WBC 7.0 x 103/mm3 4.5 - 11 RBC 6.5 x 106/μl 4.6 - 6.2 (male) Hematocrit 58 % 40 - 54 (male) MCV 68 μ3 80 - 100 RDW 14.9 % 11.6 - 14.6 Platelets 250,000 mm3 150,000 - 450,000 On examination blood pressure is 156/88 mm Hg, pulse is 76/min, temperature 98.6° F, and respirations 18/minute. In the course of your workup, an elevated erythropoietin level would exclude which diagnosis as an etiology of erythrocytosis? 1 Polycythemia vera 2 Renal tumor 3 Hydronephrosis 4 Chronic obstructive pulmonary disease 5 Sleep apnea

1 Polycythemia vera

A 1-year-old boy appears to be physically underdeveloped. His vital signs are not remarkable. His parents are concerned about the boy's health because he seems to be weak and lethargic. The parents are recent immigrants from southern Italy. A brief history check reveals that both parents have a history of mild anemia. The child is afebrile. A complete blood count (CBC) is ordered and the following results were obtained: red blood cell (RBC) count of 3.0x 1012/l; Hb of 8.0 gm/dl; Hct of 24%; peripheral smear showed 1+ basophilic stippling; hypochromia; and the presence of codocytes. Based on the clinical presentation, medical history of the parents, and the CBC results, what is the most likely diagnosis? 1 Thalassemia 2 Aplastic anemia 3 Megaloblastic anemia 4 Paroxysmal nocturnal hemoglobinuria 5 Sickle cell anemia

1 Thalassemia

A 70-year-old African American male was admitted to the hospital with complaints of a high grade fever, nausea, infection and severe back pain. Nearly two years prior while walking down his steps, he fell without incurring any apparent fractures. Since then he has been complaining of mild back pain, which worsened during cough. Lab reports reveal low red blood cell count but normal neutrophil and platelet count. The serum protein electrophoresis reveals the presence of paraprotein. Bone marrow biopsy reveals 50% infiltration by plasma cells. Based on this information, what would be your diagnosis? 1 multiple myeloma 2 Waldenström's macroglobulinemia 3 monoclonal gammopathy of unknown significance (MGUS) 4 primary amyloidosis 5 polyclonal hypergammaglobulinemia

1 multiple myeloma

A 45-year-old woman presents with a large rash on her right breast. The patient states that this black patch appeared a few days prior to presentation, and it is very painful. There are no other rashes. The patient underwent surgery for a hip fracture a few months prior to presentation. She developed a pulmonary embolism due to a deep vein thrombosis during her post-operative course. She was treated with warfarin, and she has been on the medication for the past few weeks. Physical examination reveals a large erythematous-purpuric lesion on her right breast with mild edema. This patient's rash is seen in the image. Labs reveal: Hb 12.5 g/dL HCT 40% WBC 10,000 cells/mm3 Platelets 240,000/mL Na 125 mEq/L K 4.1 mEq/L Cl 100 mEq/L Glucose 90 mg/dL BUN 18 mg/dL Creatinine 0.8 mg/dL What is the mechanism of the patient's skin rash? 1 Inhibition of Factor V 2 Inhibition of Protein C 3 Inhibition of Factor V Leiden 4 Inhibition of Factors II, V, and VIII 5 Inhibition of antithrombin III

2 Inhibition of Protein C

A 23-year-old African American male presents to the emergency room with acute chest pain and shortness of breath of a few hours duration. He is well known in the ER by the physicians and the paramedical staff due to his frequent visits in the past. He is found to have a temperature of 100.3 F, heart rate of 106/min, BP of 110/68, and a SPO2 of 90%. Chest auscultation shows bilateral crackles with a few wheezes. Labs reveal a Hb of 7g/dl, MCV 92, and a bilirubin of 3.1. EKG shows sinus tachycardia with normal cardiac enzymes. CXR reveals bilateral pulmonary infiltrates with minimal pleural effusion. Which of the following is the most likely diagnosis? 1 Hereditary Spherocytosis 2 Sickle cell anemia 3 Porphyria 4 G6PD deficiency 5 Autoimmune hemolytic anemia

2 Sickle cell anemia

What are features of plasma cell dyscrasia (paraproteinemia or monoclonal gammopathies)? 1 Lymphadenopathy and hepatosplenomegaly are more common in multiple myeloma 2 Presence of M spike is always an indicator of malignant plasma cell proliferation 3 Amyloidosis is an important complication 4 All patients with multiple myeloma show Bence-Jones proteins in the urine (M spike) 5 Proteins formed in plasma cell dyscrasia are the product of multiple clones of lymphoid cells

3 Amyloidosis is an important complication

A 17-year-old boy presents with a 3-week history of severe, worsening fatigue. Physical examination reveals new onset wheezing and abdominal pain. Diagnostic labs reveal severe hyperuricemia and elevated serum lactate dehydrogenase. Based on the history, physical, and other studies, a diagnosis of lymphoblastic lymphoma is made for this patient. The combination chemotherapy this patient will undergo is based on the regimen for what other malignant process? 1 Acute lymphoblastic leukemia (ALL) 2 Acute myelogenous leukemia (AML) 3 Chronic lymphoblastic leukemia (CLL) 4 Chronic myelogenous leukemia (CML) 5 Anaplastic large cell lymphoma

1 Acute lymphoblastic leukemia (ALL)

A 41-year-old woman is found to have anemia with Hb of 10g/dl on a routine physical exam and labs. Patient has a history of hypercholesterolemia, depression, and rheumatoid arthritis. She is on a low cholesterol diet, escitalopram (Lexapro) 10 mg daily, and diclofenac 50mg twice daily. Physical examination was essentially unremarkable except for mild arthritic changes at the proximal interphalangeal joints of both hands. Further labs reveal normal MCV and MCHC on peripheral smear; serum iron and TIBC levels are low; percentage saturation with iron is normal; and ferritin level is moderately high. Which is the most likely condition? 1 Anemia of chronic disease 2 Pernicious anemia 3 Thalassemia 4 Iron deficiency anemia 5 Aplastic anemia

1 Anemia of chronic disease

A 24-year-old woman notices that she has a bruising tendency. She frequently has numerous small bruises and purple blotches on her skin. She hates having dental work because of the associated bleeding. On physical exam, you note that she has numerous petechiae. Her lab results are shown in the chart. TEST RESULTS REFERENCE RANGE Hematocrit 42 40 - 50 (male) 36 - 44 (female) Hemoglobin 14 gm/dL 13.8 - 17.2 gm/dL (male) 12.1 - 15.1 gm/dL (female) Platelets 41,000/mm3 130,000-400,000/mm3 PT 11.5 seconds 10.8 - 13.0 seconds PTT 30 seconds 25 - 41 seconds RBC 4.8 x 106/µl 4.5 - 5.7 x 106/µl (male) 3.9 - 5.0 x 106/µl (female) The patient's condition is most likely to be characterized by what? 1 Anti platelet IgG 2 Decreased number of megakaryocytes 3 Spontaneous resolution 4 Requiring confirmation by bone marrow biopsy 5 Hemorrhagic death likely

1 Anti platelet IgG

A 65-year-old male presents to the emergency room following initiation of chemotherapy for a recent diagnosis of lymphoma. His past medical history is significant for chronic renal insufficiency. Over the past few days, he has been experiencing vomiting, restlessness, muscle weakness, and tingling sensations. On physical examination, his mucous membranes are dry. His vitals reveal a blood pressure of 90/40 mmHg and a heart rate of 110/min. Lab results reveal a white blood cell count of 30,000 cells/mm3, BUN 38 mg/dl, Creatinine 2.2 mg/dl, potassium 5.0 mEq/l, and calcium of 6.0mg/dL. Which of the following therapies is the most crucial in managing this patient? 1 IV hydration 2 Hemodialysis 3 Sodium bicarbonate 4 Sodium polystyrene sulfonate 5 Calcium gluconate

1 IV hydration

A 19-year-old woman presents with weakness and increased fatigue over the past several months. Laboratory studies reveal a hemoglobin of 9.7g/dL with a hematocrit of 32%. Which of the following results would indicate an iron deficiency anemia? 1 Increased TIBC, decreased serum ferritin, decreased serum iron 2 Decreased TIBC, decreased serum ferritin, decreased serum iron 3 Increased TIBC, increased serum ferritin, increased serum iron 4 Increased TIBC, increased serum ferritin, decreased serum iron 5 Decreased TIBC, increased serum ferritin, decreased serum iron

1 Increased TIBC, decreased serum ferritin, decreased serum iron

While covering the ED, you attend to a 12-year-old boy who cut his hand while trying to slice a bagel. His mother wrapped his hand with a dishrag and drove him to the ED. On exam, his vitals are stable; he has remained alert and cooperative but the rag has soaked through; his fingers feel numb. On removal of the rag, there is a fairly deep looking straight-edged, transverse laceration running the length of his palm, with overlying clotted blood that begins to actively bleed. In overall wound management, what is the first priority in the evaluation in this patient? 1 Debridement of any devitalized tissue 2 Thorough wound cleansing with removal of debris 3 Hemostasis 4 Thorough history of the wound mechanism and patient health including tetanus immunization history 5 Early closure of the wound

3 Hemostasis

Which of the following electrolyte imbalances occurs most frequently in patients with cancer? 1 Hypokalemia 2 Hyponatremia 3 Hypercalcemia 4 Hypomagnesemia 5 Hypocalcemia

3 Hypercalcemia

A 42-year-old African-American woman presents with increased fatigue and unusual cravings for chalk and raw potatoes over the past few months. What is the most likely diagnosis? 1 Pernicious anemia 2 Hemolytic anemia 3 Sickle cell anemia 4 Iron deficiency anemia 5 Anemia of chronic disease

4 Iron deficiency anemia

A 70-year-old male complains of insidious onset of shortness of breath with fatigue and poor energy. He is otherwise healthy and on no medications. He denies fever or recent infection. On examination, he has pallor, no lymphadenopathy, and no hepatosplenomegaly or neurological deficits. He is afebrile, BP is 130/80 mm Hg, pulse is 76/minute and respiratory rate is 16/minute. Labs show Hb 8.0g/dl, hematocrit 23%, WBC 1800/uL, platelets 56,000/uL and reticulocyte count of 0.9%.What is the most likely diagnosis? 1 Aplastic anemia 2 Chronic lymphocytic leukemia 3 Non Hodgkin's lymphoma 4 Myelodysplastic Syndrome 5 Anemia of chronic disease

4 Myelodysplastic Syndrome

You are notified that the paramedics are en route with an 8-year-old boy who was involved in a car vs. pedestrian accident. He has evidence of a femur fracture and possible internal injuries. He was intubated in the field and IV access was obtained to provide oxygen and vigorous fluid resuscitation begun. In the ED, as he is being assessed for further injuries, he is given several transfusions of packed red blood cells for substantial blood loss, but bleeding continues. What additional blood products should now be given? 1 Factor VIII concentrate 2 Cryoprecipitate 3 Washed RBCs 4 Platelets and clotting factors 5 Albumin

4 Platelets and clotting factors

A 3-year-old African-American girl with sickle cell disease presents with a limp; her mother first noticed the limp yesterday morning. The girl has not had any fever, nausea, or vomiting, but she in pain. The girl's sickle cell disease has been relatively mild; she is fully immunized and takes prophylactic oral penicillin 125 mg 2 times a day. On exam, she is afebrile. There is pain in the left hip on internal and external rotation, but there is no definite point tenderness. Exam of the left knee and right hip are negative. You are concerned about septic arthritis, and you plan further work up. What organism are you most concerned about? 1 Staphylococcus aureus 2 Streptococcus species 3 Haemophilus influenzae type b 4 Salmonella species 5 Neisseria gonorrhoeae

4 Salmonella species

A 73-year-old woman is evaluated with severe lower back and shoulder pain. As it is her 2nd visit for these symptoms in the past 1 month, she is admitted for further evaluation. She was previously healthy; she was taking daily calcium with vitamin D for mild osteoporosis. She is found to be anemic; there are multiple sites of bone tenderness. Lab values show hypercalcemia; total immunoglobulin level is raised. An X-ray of her humerus is shown in the image. A decision is made in conjunction with her medical team, and she is declared fit to undergo treatment. What is the next best step in therapy? 1 Bone marrow transplantation 2 Imatinib 3 Methotrexate 4 External beam radiation 5 Bortezomib

5 Bortezomib


Ensembles d'études connexes

Chapter 9 Sterilization Procedures, Instrument Indentification and Surgical Supplies

View Set

Kin Nutrition Cypress College Chapter # 6

View Set

World geography chapter 7 CANADA

View Set

Finance 300 Practice Questions Chapter 7

View Set

5.2.2 Quiz: Public Policy and Your Safety

View Set

Anatomy of the Sacrum and Coccyx

View Set

POM exam 2 layla lunde chapter 5

View Set